- PowerScore Staff
- Posts: 5972
- Joined: Mar 25, 2011
- Sat Jun 11, 2016 12:21 pm
#26444
Complete Question Explanation
(The complete setup for this game can be found here: lsat/viewtopic.php?t=8639)
The correct answer choice is (C)
Apply the rules and inferences in this order: inference that H must be first, fifth rule, second rule, and finally, the first, third, and fourth rules can be applied in any order since they are roughly equivalent in form.
Answer choice (A) is incorrect because H must be more popular than J.
Answer choice (B) is incorrect because J must be more popular than Q.
Answer choice (C) is the correct answer choice.
Answer choice (D) is incorrect because L must be more popular than V.
Answer choice (E) is incorrect because S cannot be seventh.
(The complete setup for this game can be found here: lsat/viewtopic.php?t=8639)
The correct answer choice is (C)
Apply the rules and inferences in this order: inference that H must be first, fifth rule, second rule, and finally, the first, third, and fourth rules can be applied in any order since they are roughly equivalent in form.
Answer choice (A) is incorrect because H must be more popular than J.
Answer choice (B) is incorrect because J must be more popular than Q.
Answer choice (C) is the correct answer choice.
Answer choice (D) is incorrect because L must be more popular than V.
Answer choice (E) is incorrect because S cannot be seventh.
Dave Killoran
PowerScore Test Preparation
Follow me on X/Twitter at http://twitter.com/DaveKilloran
My LSAT Articles: http://blog.powerscore.com/lsat/author/dave-killoran
PowerScore Podcast: http://www.powerscore.com/lsat/podcast/
PowerScore Test Preparation
Follow me on X/Twitter at http://twitter.com/DaveKilloran
My LSAT Articles: http://blog.powerscore.com/lsat/author/dave-killoran
PowerScore Podcast: http://www.powerscore.com/lsat/podcast/